Những câu hỏi liên quan
Hiền Nguyễn Thu
Xem chi tiết
truong nhat  linh
18 tháng 6 2017 lúc 22:34

1/ Ta có : tất cả các p/s ở tổng A đều có tử bằng 1 . Mà MS 101 < 102 ; 103 ; ... ; < 200 .

   Nên 1/101 là p/s lớn nhất ( lớn hơn 1/102 ; 1/103 ; ... ; 1/200 )

2/ Tổng A có phân số là : ( 200 - 101 ) : 1 + 1 = 100 (phân số ) .

Nếu thay cả 100 p/s bằng p/s lớn nhất : 1/101 thì tổng A = 1/101 . 100 = 100/101 < 1 .

=> 1/101 + 1/102 + 1/103 + ... + 1/200 ( 100p/s ) < 1/101 + 1/101 + 1/101 + ... + 1/101 (100 p/s ) < 1 .

Vậy : A < 1

Bình luận (0)
Nguyễn Đức Anh
16 tháng 3 2022 lúc 17:22
Đúng rồi
Bình luận (0)
 Khách vãng lai đã xóa
Nguyễn Đức Anh
16 tháng 3 2022 lúc 17:23
Sai sai rồi
Bình luận (0)
 Khách vãng lai đã xóa
Phạm Ngọc Linh
Xem chi tiết
Xyz OLM
3 tháng 2 2023 lúc 12:55

c) P = \(\dfrac{1}{101}+\dfrac{1}{102}+\dfrac{1}{103}+...+\dfrac{1}{200}\)

\(=\left(\dfrac{1}{101}+\dfrac{1}{102}+...+\dfrac{1}{150}\right)+\left(\dfrac{1}{151}+\dfrac{1}{152}+...+\dfrac{1}{200}\right)\)

Dễ thấy \(\dfrac{1}{101}+\dfrac{1}{102}+...+\dfrac{1}{150}>\dfrac{1}{150}+\dfrac{1}{150}+...+\dfrac{1}{150}\)(50 hạng tử)

\(\Leftrightarrow\dfrac{1}{101}+\dfrac{1}{102}+...+\dfrac{1}{150}>\dfrac{1}{150}.50=\dfrac{1}{3}\)(1)

Tương tự

 \(\dfrac{1}{151}+\dfrac{1}{152}+...+\dfrac{1}{200}>\dfrac{1}{200}+\dfrac{1}{200}+...+\dfrac{1}{200}\)(50 hạng tử)

\(\Leftrightarrow\dfrac{1}{151}+\dfrac{1}{152}+...+\dfrac{1}{200}>50.\dfrac{1}{200}=\dfrac{1}{4}\)(2) 

Từ (1) và (2) ta được

\(P>\dfrac{1}{3}+\dfrac{1}{4}=\dfrac{7}{12}\) 

Bình luận (0)
Xyz OLM
3 tháng 2 2023 lúc 13:08

P = \(\dfrac{1}{101}+\dfrac{1}{102}+\dfrac{1}{103}+...+\dfrac{1}{200}\)

\(=\left(\dfrac{1}{101}+\dfrac{1}{102}+...+\dfrac{1}{150}\right)+\left(\dfrac{1}{151}+\dfrac{1}{152}+...+\dfrac{1}{200}\right)\)

         \(\overline{50\text{ hạng tử }}\)                            \(\overline{50\text{ hạng tử }}\)

\(< \left(\dfrac{1}{100}+\dfrac{1}{100}+...+\dfrac{1}{100}\right)+\left(\dfrac{1}{150}+\dfrac{1}{150}+...+\dfrac{1}{150}\right)\) 

\(=\dfrac{1}{100}.50+\dfrac{1}{150}.50=\dfrac{1}{2}+\dfrac{1}{3}=\dfrac{5}{6}\)

\(\Rightarrow P< \dfrac{5}{6}< 1\)

Bình luận (0)
Lê Nhi
Xem chi tiết
Long Sơn
Xem chi tiết
chuche
25 tháng 3 2022 lúc 20:15

M>N

Bình luận (0)
Minh Hiếu
25 tháng 3 2022 lúc 20:17

Tham khảo:

https://hoc247.net/hoi-dap/toan-6/so-sanh-m-101-102-1-101-103-1-va-n-101-103-1-101-104-1--faq225210.html

Bình luận (0)
TV Cuber
25 tháng 3 2022 lúc 20:17

\(M>N\)

Bình luận (0)
Phạm Hoàng Nam
Xem chi tiết
Sky Love MTP
14 tháng 2 2016 lúc 20:36

j mà  nhìu zu zậy làm bao giờ mới xong

Bình luận (0)
Trần Thanh Phương
14 tháng 2 2016 lúc 20:38

Ủng hộ mk đi các bạn
 

Bình luận (0)
Trần Thuỳ Trang
Xem chi tiết
Lê Tài Bảo Châu
3 tháng 5 2019 lúc 21:34

b) Ta có: \(\frac{1}{101}>0\)

              \(\frac{1}{102}>0\)

                ...............,....

                 \(\frac{1}{200}>0\)

\(\Rightarrow S>0\left(1\right)\)

Lại có: \(\frac{1}{101}< \frac{1}{100}\)

             \(\frac{1}{102}< \frac{1}{100}\)

               ......................

             \(\frac{1}{200}< \frac{1}{100}\)

\(\Rightarrow S< \frac{1}{100}.100\)

\(\Rightarrow S< 1\left(2\right)\)

Từ (1) và (2) \(\Rightarrow0< S< 1\)

Vậy S ko là   số tự nhiên

Bình luận (0)
Trần Tuấn Anh
3 tháng 5 2019 lúc 21:35

a, ta có 1/101<1/100; 1/102<1/100;...;1/109<1/100

=> S=1/101+1/102+...+1/109< 1/100+1/100+...+1/100=9/100

=>S<9/100

b,ta thấy S luôn >0

S=1/101+1/102+...+1/200<1/100+1/100+...+1/100=1

=>S<1

=>0<S<1 => S không phải số tự nhiên

Bình luận (0)
zZz Cool Kid_new zZz
3 tháng 5 2019 lúc 21:49

\(\frac{1}{101}< \frac{1}{100};\frac{1}{102}< \frac{1}{100};\frac{1}{103}< \frac{1}{100};......;\frac{1}{109}< \frac{1}{100}\)

\(\Rightarrow\frac{1}{101}+\frac{1}{102}+\frac{1}{103}+....+\frac{1}{109}< \frac{1}{100}+\frac{1}{100}+\frac{1}{100}+....+\frac{1}{100}\)

\(\Rightarrow S< 9\cdot\frac{1}{100}\)

\(\Rightarrow S< \frac{9}{100}\)

Vậy \(S< \frac{9}{100}\)

Bình luận (0)
Trần Thuỳ Trang7897
Xem chi tiết

Có:\(\frac{1}{101}< \frac{1}{100}\)

     \(\frac{1}{102}< \frac{1}{100}\)

      ........................

      \(\frac{1}{109}< \frac{1}{100}\)

=>\(\frac{1}{101}+\frac{1}{102}+...+\frac{1}{109}< \frac{1}{100}+\frac{1}{100}+...+\frac{1}{100}\)

                                                                               (9 phân số)

\(=>\frac{1}{101}+\frac{1}{102}+...+\frac{1}{109}< \frac{9}{100}\)

Bình luận (0)
Bùi Ngọc Minh Hà
Xem chi tiết
Bùi Ngọc Minh Hà
16 tháng 4 2022 lúc 14:30

giải giúp mink với

 

Bình luận (0)
Cihce
16 tháng 4 2022 lúc 14:31

M > N

Bình luận (0)
Nguyen minh Chien
Xem chi tiết